LSAT and Law School Admissions Forum

Get expert LSAT preparation and law school admissions advice from PowerScore Test Preparation.

 rohall
  • Posts: 3
  • Joined: Sep 11, 2018
|
#57956
Steve Stein wrote:Hi Nadia,

Thanks for your follow-up. The first sentence provides that it is wrong to restrict liberty, except perhaps when failing to do so would allow harm. In other words, if it is not wrong to restrict liberty, we know that failing to do so would allow harm:

Wrong to restrict liberty :arrow: failing to restrict would allow for harm

...the contrapositive of the above statement provides that if failing to restrict something would not allow for harm, it would be wrong to restrict liberty:

failing to restrict would allow for harm :arrow: Wrong to restrict liberty

Choice (A) works with the statement above for confirmation: if a publication is only offensive, then failing to restrict it would not allow for harm, so it would be wrong to restrict liberty in that scenario:

merely offensive :arrow: failing to restrict would allow for harm :arrow: Wrong to restrict liberty

Answer choice (E) can be ruled out, because, as you point out, "serious" is not supported by the stimulus.

I hope that's helpful! Please let me know whether this is clear--thanks!

~Steve
I'm confused about this one because don't we need to negate "It is wrong" to "it is not wrong" in the first sentence...given the except rule?
 Rachael Wilkenfeld
PowerScore Staff
  • PowerScore Staff
  • Posts: 1392
  • Joined: Dec 15, 2011
|
#57979
Hi Rohall,

You are correct. The first line of your quote shows " wrong to restrict liberty" as underlined when it should look crossed out. But the contrapositive below shows the correct relationship. not failing to restrict would allow for harm :arrow: wrong to restrict liberty.

Hope that helps!
 michellesdesh
  • Posts: 2
  • Joined: Jul 13, 2020
|
#77142
*Saw that the hw explanation was locked but still confused so opening this up

I narrowed the choices to A and B but ended up choosing B instead of A. My confusion lies in the application of the rule introduced in L2 about the four words [unless/except/until/without], which is to put the term immediately after one of those words on the right side of the arrow (necessary term), and to negate the other condition and put it on the sufficient term side. In this case, I diagrammed the following from the first sentence's use of "except":

NOT wrong for the government to restrict the liberty of individuals (negated the other condition) :arrow: when failing to do so would allow individuals to cause harm (put the term immediately following "except" on the right side)

In this case, wouldn't B just be that sentence word for word? Maybe I understood the rule wrong, but it looks like answer B is the exact sentence for how I would apply that rule with the words [unless/except/until/without]. Please help, thanks!
User avatar
 Stephanie Oswalt
PowerScore Staff
  • PowerScore Staff
  • Posts: 826
  • Joined: Jan 11, 2016
|
#77177
Hi Michelle,

I moved your question to the thread discussing this topic, and I've temporarily unlocked it. There are dozens of posts about this question, so we chose to lock the thread to prevent the topic from becoming even more unwieldy than it already is. :-D

I would recommend first looking through this thread to see if any prior explanations help. Particularly, please refer to page 1, the convo between Steve Stein and hanvan:
Steve Stein
Thanks for your question. "Except perhaps" means that there might be an exception--under certain conditions. That is, certain necessary conditions.

When the author says that it is wrong to restrict individual liberties, except perhaps (in some limited cases) to prevent harm.

In other words, the only situation in which such a restriction would be warranted (if ever) would be to prevent harm:

restriction of liberty not wrong --> prevent harm

Let me know whether that clears this one up--thanks!

~Steve
hanvan
I looked again in the answer B, it is Mistaken Reversal, because of the phrase " it is not wrong.....when failing to do so...". That's why this answer choice is wrong.
Thanks, Steve.
Also, on page 4, please refer to Jon's post:
Jon

Answer choice (B) for this one does actually use conditional language, as the word "when" is a sufficient condition indicator (functions similarly to "if"). So if we diagrammed (B), we'd get something like:

..... Harm :arrow: Not Wrong Restrict Liberty

And the reason that's a loose Mistaken Reversal--I wouldn't call it a perfect MR because, as Robert mentions, the first sentence isn't perfectly absolute the way most conditional reasoning is (the word "perhaps" softens it a bit)--is because if we diagrammed the first sentence conditionally we'd show it as:

..... Not Wrong Restrict Liberty :arrow: Harm

where knowing that it was not wrong (or permissible) to actually restrict individual liberty would tell you that doing so must allow those individuals to cause harm. Put another way, if liberty is going to be restricted, you know the one possible reason to restrict it--the exception to freedom: cause harm--must occur.

So in that sense (B) does reverse the ideas from the first sentence, and would generally fall under the category of a Mistaken Reversal.

I hope that helps!

Jon
Also, please refer to nicholaspavic's post on page 7:
nicholaspavic
This is always a tricky question for students in their review. But let's take a close look at Answer Choice (B) and give it a good analysis.

Answer choice (B) uses conditional language, as the word "when" is a sufficient condition indicator ("when" functions similarly to "if"). So when we diagram (B), it will look like this:

..... Harm :arrow: Not Wrong Restrict Liberty

That's a loose Mistaken Reversal of the statement that we have in the stimulus because the stimulus' first sentence isn't "perfectly phrased" conditional reasoning (the word "perhaps" softens the language). But when we diagram this soft statement, it looks like this:

..... Not Wrong Restrict Liberty :arrow: Harm

So with this stimulus' statement, knowing that it was not wrong (or permissible) to actually restrict individual liberty would tell you that doing so must allow those individuals to cause harm. Put another way, if liberty is going to be restricted, you know the one possible reason to restrict it--the exception to freedom: cause harm--must occur.

So in that sense Answer choice (B) does reverse the ideas from the first sentence, and would generally fall under the category of a Mistaken Reversal as discussed in our course.

Please let us know if that helps and welcome again!
There are several other explanations scattered throughout this thread that reference answer choice (B). If, after reviewing these you still are confused, please let us know. :)

Thanks!
 g_lawyered
  • Posts: 213
  • Joined: Sep 14, 2020
|
#79148
Hello,
I clicked on the link that the original post had but was confused when I read the explanation. I was reading Steve's explanation and this part of his reasoning didn't click with me. Especially about the exception to preventing harm, like how do we write that out as conditional statement? :-?

This is what Steve posted:
Synopsis of stimulus:

If not to prevent harm -----> Gov't restricting liberty of individuals is wrong.
Publish something ---> liberty
Offend ----> not harm

Question type: Must Be True

Prephrase: What do we know for sure from the information above? Let's connect the statements. Since we know publishing something is a liberty, we therefore know Gov't restricting individuals' publishing something is wrong, unless maybe to prevent harm.

We also know to offend is not harm. Therefore we can further connect the dots to establish that Gov't restricting individuals' publishing something that is only offensive is wrong.

This is because the one case in which the government might be justified in restricting a liberty is when such a liberty could cause harm to others. To offend fails to pass this test.

Can someone explain why answer choice A is correct?

Thanks in advance!
User avatar
 Stephanie Oswalt
PowerScore Staff
  • PowerScore Staff
  • Posts: 826
  • Joined: Jan 11, 2016
|
#79154
GGIBA003@FIU.EDU wrote:Hello,
I clicked on the link that the original post had but was confused when I read the explanation. I was reading Steve's explanation ....
Can someone explain why answer choice A is correct?

Thanks in advance!
Hi GGIBA003,

Thanks for the post!

We have locked this particular thread because it is rather lengthy and full of explanations, so we'd prefer to keep this thread from becoming even more of a behemoth than it already is. :-D Please review Jonathan's explanation on page 1, and closely look through the following discussion (there's a lot here already!!) to see if that helps! Here's a few highlights that may help:


A) This is the credited response. This is a spot on match for our prephrase. Since we know to offend is not to cause harm, we know that restricting liberty of something that is only offensive would not meet the criteria established in the stimulus. This passes the Fact Test™ and is correct.
Answer choice (A) tells us it is not right for the government to restrict the publication of literature that is only offensive. "Not right" = "wrong" and the stimulus already told us that publishing is a liberty. We know that restricting liberty is wrong in all cases where there is no chance of causing harm, and the stimulus told us that giving offense is not causing harm. Is there any possible other harm that could be caused? No, because this answer choice says the publication is "only offensive." There is not other possible effect, so no other possible cause of harm to consider.
Answer choice (A): Let's take this one step at a time. What do we know about the publication of literature? It is a liberty. What do we know about liberties? It is wrong for the government to restrict them unless perhaps they cause harm. Therefore, given that publication does not cause harm, it is categorically wrong to restrict such a liberty. Now, connect the other information. What do we know about something that is only offensive? It does not cause harm. Therefore we know that this particular publication is a liberty that does not cause harm. Thus, it would be wrong to restrict such publication.

Return to our explanation of what it would mean for something to be wrong:

"it is not right" :dbl: (morally neutral OR "it is wrong")

Thus, to be wrong is in fact a sufficient condition to know something "is not right"

"it is wrong" :arrow: "it is not right"

Thus, (A) is supported.
Answer choice (A) uses the word "only," so it is tempting to diagram it as a conditional. However, what it's really saying is that "It is not right for the government to restrict the publication of literature that is only offensive does not cause harm." It's just like if you said "I only had a bologna sandwich for lunch." That means you did not have a PBJ or a ham sandwich or any other sandwich. Thus, answer choice (A) is talking about publication (liberty!) that does not cause harm (not an exception to the rule!). And if we apply the rule, then it would be wrong to restrict that liberty.
This question actually only contains one conditional statment, in the first sentence:

Wrong to Restrict Liberty (WRL) :arrow: Cause Harm (CH)

and its contrapositive:

CH :arrow: WRL

Then it gives us specific examples that would fall under the general conditions given:

Publish = Liberty

Offend = Cause Harm

From this, we can see we have to apply the contrapositive to the situation, which gives us:

CHOffend :arrow: WRLPublish

Meaning that it would be wrong to restrict publication of offensive material. Answer choice (A) reflects this, making it the correct answer.

Thanks!
  • 1
  • 4
  • 5
  • 6
  • 7
  • 8

Get the most out of your LSAT Prep Plus subscription.

Analyze and track your performance with our Testing and Analytics Package.